Prove that there is no integer a for which $a^2 - 3a - 19$ is divisible by 289












2












$begingroup$


Prove that there is no integer a for which $a^2 - 3a - 19$ is divisible by 289.
Not got any clue. Please help.










share|cite|improve this question









$endgroup$












  • $begingroup$
    HINT: Check divisibility by $17$
    $endgroup$
    – lab bhattacharjee
    Nov 30 '13 at 10:55










  • $begingroup$
    And how to check divisibility by 17 ?
    $endgroup$
    – user2369284
    Nov 30 '13 at 10:56
















2












$begingroup$


Prove that there is no integer a for which $a^2 - 3a - 19$ is divisible by 289.
Not got any clue. Please help.










share|cite|improve this question









$endgroup$












  • $begingroup$
    HINT: Check divisibility by $17$
    $endgroup$
    – lab bhattacharjee
    Nov 30 '13 at 10:55










  • $begingroup$
    And how to check divisibility by 17 ?
    $endgroup$
    – user2369284
    Nov 30 '13 at 10:56














2












2








2


3



$begingroup$


Prove that there is no integer a for which $a^2 - 3a - 19$ is divisible by 289.
Not got any clue. Please help.










share|cite|improve this question









$endgroup$




Prove that there is no integer a for which $a^2 - 3a - 19$ is divisible by 289.
Not got any clue. Please help.







elementary-number-theory






share|cite|improve this question













share|cite|improve this question











share|cite|improve this question




share|cite|improve this question










asked Nov 30 '13 at 10:47









user2369284user2369284

1,403927




1,403927












  • $begingroup$
    HINT: Check divisibility by $17$
    $endgroup$
    – lab bhattacharjee
    Nov 30 '13 at 10:55










  • $begingroup$
    And how to check divisibility by 17 ?
    $endgroup$
    – user2369284
    Nov 30 '13 at 10:56


















  • $begingroup$
    HINT: Check divisibility by $17$
    $endgroup$
    – lab bhattacharjee
    Nov 30 '13 at 10:55










  • $begingroup$
    And how to check divisibility by 17 ?
    $endgroup$
    – user2369284
    Nov 30 '13 at 10:56
















$begingroup$
HINT: Check divisibility by $17$
$endgroup$
– lab bhattacharjee
Nov 30 '13 at 10:55




$begingroup$
HINT: Check divisibility by $17$
$endgroup$
– lab bhattacharjee
Nov 30 '13 at 10:55












$begingroup$
And how to check divisibility by 17 ?
$endgroup$
– user2369284
Nov 30 '13 at 10:56




$begingroup$
And how to check divisibility by 17 ?
$endgroup$
– user2369284
Nov 30 '13 at 10:56










5 Answers
5






active

oldest

votes


















6












$begingroup$

Assume that $a^2 -3a -19 = 289n$ for some integers $a$, and $n$.



Then: $a^2 - 3a -19 - 289n = 0$, taking "delta"



$$Delta = b^2 - 4ac = (-3)^2 -4(1)(-19-289n) = 9 + 76 + 1156n = 85 + 1156n = 17(5 + 4times17n)$$



must be a perfect square.



Thus $17(5 + 4 times 17n)$ can be written as a product of squares of primes. Since $17$ is a prime itself, the factor $5 + 4 times 17n = 17p^2 times q^2 times ...t^2$ with $p, q, ..., t$ are primes.



But this means $5 + 4 times 17n = 0 pmod{17}$ a contradiction since it is $5 pmod{17}$.






share|cite|improve this answer











$endgroup$













  • $begingroup$
    great answer!!!
    $endgroup$
    – user2369284
    Nov 30 '13 at 11:38






  • 1




    $begingroup$
    The expression that must be a perfect square is $17(5+4*17n)$, not $17(5+4*289n)$. It doesn't affect the argument, though.
    $endgroup$
    – Barry Cipra
    Nov 30 '13 at 12:38



















7












$begingroup$

$a^2-3a-19 = (a+7)^2-17(a+4)$. For $17^2$ to divide this, $17$ has to divide both $a+7$ and $a+4$, which is impossible.






share|cite|improve this answer









$endgroup$





















    2












    $begingroup$

    Doing algebra with number systems is often easier than trying to do algebra with divisibility. This is one of those cases.



    What you are asked to show is the same thing as showing



    $$ a^2 -3a - 19 equiv 0 bmod 289 $$



    has no solutions. $289$ is odd so that we can divide by $2$ — in particular, that means we can solve quadratic equations in the usual way. You can go through the exercise of completing the square if you like, but the quadratic formula works too:



    $$begin{align}
    a &= frac{3 + sqrt{(-3)^2 - 4 cdot (-19)}}{2} mod{289 }
    \ &= frac{3 + sqrt{85}}{2} mod{289}
    end{align}
    $$



    where each choice of square root gives a solution, and conversely, every solution comes from a choice of square root. (for general moduli, it is possible to have more than two square roots, although that can't happen for odd prime powers, such as this problem)



    To compute square roots, it is usually best to work modulo the prime factors first, then lift to powers of primes, and finally use the Chinese Remainder Theorem to patch the solution together. Here, $289 = 17^2$, so we only have to do the first two steps.



    So first, we need



    $$ sqrt{85} = sqrt{0} = 0 mod{17}$$



    Thus, if $85$ has a square root modulo $289$, it must be a multiple of 17.



    However,



    $$ (17c)^2 = 289c^2 = 0 mod{289} $$



    therefore $85$ does not have a square root modulo $289$. Thus, there is no solution for $a$.






    share|cite|improve this answer











    $endgroup$





















      1












      $begingroup$

      As $289=17^2,$ let us check the divisibility by $17$



      $$a^2-3a-19equiv0pmod{289}implies a^2-3a-19equiv0pmod{17}$$



      $$iff a^2-3aequiv19equiv2iff4a^2-12aequiv8pmod{17}text{ as }(4,17)=1$$



      $$iff(2a-3)^2equiv0$$



      $$iff 2aequiv3pmod{17}equiv20iff aequiv10text{ as }(2,17)=1$$



      So, $a=17c+10$ where $c$ is some integer



      $$a^2-3a-19=(17c+10)^2-3(17c+10)-19equiv51pmod{289}$$






      share|cite|improve this answer









      $endgroup$













      • $begingroup$
        Initially , have you taken the expression to be divisible by 17 ?
        $endgroup$
        – user2369284
        Nov 30 '13 at 11:16










      • $begingroup$
        @user2369284, yes, I have solved for modulo $17$ as it is a factor of $289$
        $endgroup$
        – lab bhattacharjee
        Nov 30 '13 at 11:18










      • $begingroup$
        but how can you take so when it is not given ?
        $endgroup$
        – user2369284
        Nov 30 '13 at 11:19










      • $begingroup$
        @user2369284, do you know, if $a$ divides $b$ and $b$ divides $c;a$ must divide $c$. Btw, you can check other answers
        $endgroup$
        – lab bhattacharjee
        Nov 30 '13 at 11:20










      • $begingroup$
        @user2369284: No, you don't have to start by solving modulo $17$. However, the arithmetic involved is sometimes simpler if you solve modulo 17, then lift the solution to modulo 289 (e.g. this is often done to calculate square roots). IMO, you shouldn't think about divisibility at all: you should be thinking about modular arithmetic. But if you insist, you know that something divisible by 289 must also be divisible by 17, so it's fair to start there.
        $endgroup$
        – Hurkyl
        Nov 30 '13 at 14:54





















      1












      $begingroup$

      Inspired by mercio



      let us find $x,y$ such that $x-y=3,x+y=17implies x=10,y=7$



      $$a^2-3a-19=(a-10)(a+7)+51$$



      As $51$ is divisible by $17,$ so must be $(a-10)(a+7)$ to make $17|(a^2-3a-19)$



      Now $17|(a-10)iff 17|(a+7)$ as $(a+7)-(a-10)=17$



      So, $17^2|(a-10)(a+7),$ but $17^2not|51$



      Generalization:



      Let us find integer $x,y$ such that



      $displaystyle x-y=-3$(the coefficient of $a$)



      and $displaystyle x+y=17c$ so that either both or none $(a+x),(a-y)$ will be divisible by $17$



      $implies 2x=17c-3implies c$ must be odd $=2d+1$(say) for some integer $d$



      $implies x=17d+7$ and $y=x+3=17d+10$



      Now, $displaystyle (a+x)(a+y)={a+(17d+7)}{a-(17d+10)}=a^2-3a-(17d+7)(17d+10)$



      $displaystyleimplies a^2-3a-19={a+(17d+7)}{a-(17d+10)}+(17d+7)(17d+10)-19$



      $displaystyleimplies a^2-3a-19=underbrace{{a+(17d+7)}{a-(17d+10)}}_{text{terms with difference }=17}+17^2(d^2+d)+51$



      Now the logic is exactly same as the one above



      In the above method $d=0$






      share|cite|improve this answer











      $endgroup$













      • $begingroup$
        why have you taken x-y=3, x+y=17,how did you feel that it was necesary ?
        $endgroup$
        – user2369284
        Nov 30 '13 at 11:23










      • $begingroup$
        @user2369284, observe the coefficient of $3$ and we need modulo $17$ first. So that both or none of $a-x,a+y$ be divisible by $17$. See this(math.stackexchange.com/questions/9431/…)
        $endgroup$
        – lab bhattacharjee
        Nov 30 '13 at 11:27












      • $begingroup$
        @user2369284, please have a look into the edited version
        $endgroup$
        – lab bhattacharjee
        Nov 30 '13 at 15:52













      Your Answer





      StackExchange.ifUsing("editor", function () {
      return StackExchange.using("mathjaxEditing", function () {
      StackExchange.MarkdownEditor.creationCallbacks.add(function (editor, postfix) {
      StackExchange.mathjaxEditing.prepareWmdForMathJax(editor, postfix, [["$", "$"], ["\\(","\\)"]]);
      });
      });
      }, "mathjax-editing");

      StackExchange.ready(function() {
      var channelOptions = {
      tags: "".split(" "),
      id: "69"
      };
      initTagRenderer("".split(" "), "".split(" "), channelOptions);

      StackExchange.using("externalEditor", function() {
      // Have to fire editor after snippets, if snippets enabled
      if (StackExchange.settings.snippets.snippetsEnabled) {
      StackExchange.using("snippets", function() {
      createEditor();
      });
      }
      else {
      createEditor();
      }
      });

      function createEditor() {
      StackExchange.prepareEditor({
      heartbeatType: 'answer',
      autoActivateHeartbeat: false,
      convertImagesToLinks: true,
      noModals: true,
      showLowRepImageUploadWarning: true,
      reputationToPostImages: 10,
      bindNavPrevention: true,
      postfix: "",
      imageUploader: {
      brandingHtml: "Powered by u003ca class="icon-imgur-white" href="https://imgur.com/"u003eu003c/au003e",
      contentPolicyHtml: "User contributions licensed under u003ca href="https://creativecommons.org/licenses/by-sa/3.0/"u003ecc by-sa 3.0 with attribution requiredu003c/au003e u003ca href="https://stackoverflow.com/legal/content-policy"u003e(content policy)u003c/au003e",
      allowUrls: true
      },
      noCode: true, onDemand: true,
      discardSelector: ".discard-answer"
      ,immediatelyShowMarkdownHelp:true
      });


      }
      });














      draft saved

      draft discarded


















      StackExchange.ready(
      function () {
      StackExchange.openid.initPostLogin('.new-post-login', 'https%3a%2f%2fmath.stackexchange.com%2fquestions%2f586666%2fprove-that-there-is-no-integer-a-for-which-a2-3a-19-is-divisible-by-289%23new-answer', 'question_page');
      }
      );

      Post as a guest















      Required, but never shown

























      5 Answers
      5






      active

      oldest

      votes








      5 Answers
      5






      active

      oldest

      votes









      active

      oldest

      votes






      active

      oldest

      votes









      6












      $begingroup$

      Assume that $a^2 -3a -19 = 289n$ for some integers $a$, and $n$.



      Then: $a^2 - 3a -19 - 289n = 0$, taking "delta"



      $$Delta = b^2 - 4ac = (-3)^2 -4(1)(-19-289n) = 9 + 76 + 1156n = 85 + 1156n = 17(5 + 4times17n)$$



      must be a perfect square.



      Thus $17(5 + 4 times 17n)$ can be written as a product of squares of primes. Since $17$ is a prime itself, the factor $5 + 4 times 17n = 17p^2 times q^2 times ...t^2$ with $p, q, ..., t$ are primes.



      But this means $5 + 4 times 17n = 0 pmod{17}$ a contradiction since it is $5 pmod{17}$.






      share|cite|improve this answer











      $endgroup$













      • $begingroup$
        great answer!!!
        $endgroup$
        – user2369284
        Nov 30 '13 at 11:38






      • 1




        $begingroup$
        The expression that must be a perfect square is $17(5+4*17n)$, not $17(5+4*289n)$. It doesn't affect the argument, though.
        $endgroup$
        – Barry Cipra
        Nov 30 '13 at 12:38
















      6












      $begingroup$

      Assume that $a^2 -3a -19 = 289n$ for some integers $a$, and $n$.



      Then: $a^2 - 3a -19 - 289n = 0$, taking "delta"



      $$Delta = b^2 - 4ac = (-3)^2 -4(1)(-19-289n) = 9 + 76 + 1156n = 85 + 1156n = 17(5 + 4times17n)$$



      must be a perfect square.



      Thus $17(5 + 4 times 17n)$ can be written as a product of squares of primes. Since $17$ is a prime itself, the factor $5 + 4 times 17n = 17p^2 times q^2 times ...t^2$ with $p, q, ..., t$ are primes.



      But this means $5 + 4 times 17n = 0 pmod{17}$ a contradiction since it is $5 pmod{17}$.






      share|cite|improve this answer











      $endgroup$













      • $begingroup$
        great answer!!!
        $endgroup$
        – user2369284
        Nov 30 '13 at 11:38






      • 1




        $begingroup$
        The expression that must be a perfect square is $17(5+4*17n)$, not $17(5+4*289n)$. It doesn't affect the argument, though.
        $endgroup$
        – Barry Cipra
        Nov 30 '13 at 12:38














      6












      6








      6





      $begingroup$

      Assume that $a^2 -3a -19 = 289n$ for some integers $a$, and $n$.



      Then: $a^2 - 3a -19 - 289n = 0$, taking "delta"



      $$Delta = b^2 - 4ac = (-3)^2 -4(1)(-19-289n) = 9 + 76 + 1156n = 85 + 1156n = 17(5 + 4times17n)$$



      must be a perfect square.



      Thus $17(5 + 4 times 17n)$ can be written as a product of squares of primes. Since $17$ is a prime itself, the factor $5 + 4 times 17n = 17p^2 times q^2 times ...t^2$ with $p, q, ..., t$ are primes.



      But this means $5 + 4 times 17n = 0 pmod{17}$ a contradiction since it is $5 pmod{17}$.






      share|cite|improve this answer











      $endgroup$



      Assume that $a^2 -3a -19 = 289n$ for some integers $a$, and $n$.



      Then: $a^2 - 3a -19 - 289n = 0$, taking "delta"



      $$Delta = b^2 - 4ac = (-3)^2 -4(1)(-19-289n) = 9 + 76 + 1156n = 85 + 1156n = 17(5 + 4times17n)$$



      must be a perfect square.



      Thus $17(5 + 4 times 17n)$ can be written as a product of squares of primes. Since $17$ is a prime itself, the factor $5 + 4 times 17n = 17p^2 times q^2 times ...t^2$ with $p, q, ..., t$ are primes.



      But this means $5 + 4 times 17n = 0 pmod{17}$ a contradiction since it is $5 pmod{17}$.







      share|cite|improve this answer














      share|cite|improve this answer



      share|cite|improve this answer








      edited Jan 8 at 7:11









      Yellow

      16011




      16011










      answered Nov 30 '13 at 11:29









      DeepSeaDeepSea

      71.2k54487




      71.2k54487












      • $begingroup$
        great answer!!!
        $endgroup$
        – user2369284
        Nov 30 '13 at 11:38






      • 1




        $begingroup$
        The expression that must be a perfect square is $17(5+4*17n)$, not $17(5+4*289n)$. It doesn't affect the argument, though.
        $endgroup$
        – Barry Cipra
        Nov 30 '13 at 12:38


















      • $begingroup$
        great answer!!!
        $endgroup$
        – user2369284
        Nov 30 '13 at 11:38






      • 1




        $begingroup$
        The expression that must be a perfect square is $17(5+4*17n)$, not $17(5+4*289n)$. It doesn't affect the argument, though.
        $endgroup$
        – Barry Cipra
        Nov 30 '13 at 12:38
















      $begingroup$
      great answer!!!
      $endgroup$
      – user2369284
      Nov 30 '13 at 11:38




      $begingroup$
      great answer!!!
      $endgroup$
      – user2369284
      Nov 30 '13 at 11:38




      1




      1




      $begingroup$
      The expression that must be a perfect square is $17(5+4*17n)$, not $17(5+4*289n)$. It doesn't affect the argument, though.
      $endgroup$
      – Barry Cipra
      Nov 30 '13 at 12:38




      $begingroup$
      The expression that must be a perfect square is $17(5+4*17n)$, not $17(5+4*289n)$. It doesn't affect the argument, though.
      $endgroup$
      – Barry Cipra
      Nov 30 '13 at 12:38











      7












      $begingroup$

      $a^2-3a-19 = (a+7)^2-17(a+4)$. For $17^2$ to divide this, $17$ has to divide both $a+7$ and $a+4$, which is impossible.






      share|cite|improve this answer









      $endgroup$


















        7












        $begingroup$

        $a^2-3a-19 = (a+7)^2-17(a+4)$. For $17^2$ to divide this, $17$ has to divide both $a+7$ and $a+4$, which is impossible.






        share|cite|improve this answer









        $endgroup$
















          7












          7








          7





          $begingroup$

          $a^2-3a-19 = (a+7)^2-17(a+4)$. For $17^2$ to divide this, $17$ has to divide both $a+7$ and $a+4$, which is impossible.






          share|cite|improve this answer









          $endgroup$



          $a^2-3a-19 = (a+7)^2-17(a+4)$. For $17^2$ to divide this, $17$ has to divide both $a+7$ and $a+4$, which is impossible.







          share|cite|improve this answer












          share|cite|improve this answer



          share|cite|improve this answer










          answered Nov 30 '13 at 11:07









          merciomercio

          44.6k257110




          44.6k257110























              2












              $begingroup$

              Doing algebra with number systems is often easier than trying to do algebra with divisibility. This is one of those cases.



              What you are asked to show is the same thing as showing



              $$ a^2 -3a - 19 equiv 0 bmod 289 $$



              has no solutions. $289$ is odd so that we can divide by $2$ — in particular, that means we can solve quadratic equations in the usual way. You can go through the exercise of completing the square if you like, but the quadratic formula works too:



              $$begin{align}
              a &= frac{3 + sqrt{(-3)^2 - 4 cdot (-19)}}{2} mod{289 }
              \ &= frac{3 + sqrt{85}}{2} mod{289}
              end{align}
              $$



              where each choice of square root gives a solution, and conversely, every solution comes from a choice of square root. (for general moduli, it is possible to have more than two square roots, although that can't happen for odd prime powers, such as this problem)



              To compute square roots, it is usually best to work modulo the prime factors first, then lift to powers of primes, and finally use the Chinese Remainder Theorem to patch the solution together. Here, $289 = 17^2$, so we only have to do the first two steps.



              So first, we need



              $$ sqrt{85} = sqrt{0} = 0 mod{17}$$



              Thus, if $85$ has a square root modulo $289$, it must be a multiple of 17.



              However,



              $$ (17c)^2 = 289c^2 = 0 mod{289} $$



              therefore $85$ does not have a square root modulo $289$. Thus, there is no solution for $a$.






              share|cite|improve this answer











              $endgroup$


















                2












                $begingroup$

                Doing algebra with number systems is often easier than trying to do algebra with divisibility. This is one of those cases.



                What you are asked to show is the same thing as showing



                $$ a^2 -3a - 19 equiv 0 bmod 289 $$



                has no solutions. $289$ is odd so that we can divide by $2$ — in particular, that means we can solve quadratic equations in the usual way. You can go through the exercise of completing the square if you like, but the quadratic formula works too:



                $$begin{align}
                a &= frac{3 + sqrt{(-3)^2 - 4 cdot (-19)}}{2} mod{289 }
                \ &= frac{3 + sqrt{85}}{2} mod{289}
                end{align}
                $$



                where each choice of square root gives a solution, and conversely, every solution comes from a choice of square root. (for general moduli, it is possible to have more than two square roots, although that can't happen for odd prime powers, such as this problem)



                To compute square roots, it is usually best to work modulo the prime factors first, then lift to powers of primes, and finally use the Chinese Remainder Theorem to patch the solution together. Here, $289 = 17^2$, so we only have to do the first two steps.



                So first, we need



                $$ sqrt{85} = sqrt{0} = 0 mod{17}$$



                Thus, if $85$ has a square root modulo $289$, it must be a multiple of 17.



                However,



                $$ (17c)^2 = 289c^2 = 0 mod{289} $$



                therefore $85$ does not have a square root modulo $289$. Thus, there is no solution for $a$.






                share|cite|improve this answer











                $endgroup$
















                  2












                  2








                  2





                  $begingroup$

                  Doing algebra with number systems is often easier than trying to do algebra with divisibility. This is one of those cases.



                  What you are asked to show is the same thing as showing



                  $$ a^2 -3a - 19 equiv 0 bmod 289 $$



                  has no solutions. $289$ is odd so that we can divide by $2$ — in particular, that means we can solve quadratic equations in the usual way. You can go through the exercise of completing the square if you like, but the quadratic formula works too:



                  $$begin{align}
                  a &= frac{3 + sqrt{(-3)^2 - 4 cdot (-19)}}{2} mod{289 }
                  \ &= frac{3 + sqrt{85}}{2} mod{289}
                  end{align}
                  $$



                  where each choice of square root gives a solution, and conversely, every solution comes from a choice of square root. (for general moduli, it is possible to have more than two square roots, although that can't happen for odd prime powers, such as this problem)



                  To compute square roots, it is usually best to work modulo the prime factors first, then lift to powers of primes, and finally use the Chinese Remainder Theorem to patch the solution together. Here, $289 = 17^2$, so we only have to do the first two steps.



                  So first, we need



                  $$ sqrt{85} = sqrt{0} = 0 mod{17}$$



                  Thus, if $85$ has a square root modulo $289$, it must be a multiple of 17.



                  However,



                  $$ (17c)^2 = 289c^2 = 0 mod{289} $$



                  therefore $85$ does not have a square root modulo $289$. Thus, there is no solution for $a$.






                  share|cite|improve this answer











                  $endgroup$



                  Doing algebra with number systems is often easier than trying to do algebra with divisibility. This is one of those cases.



                  What you are asked to show is the same thing as showing



                  $$ a^2 -3a - 19 equiv 0 bmod 289 $$



                  has no solutions. $289$ is odd so that we can divide by $2$ — in particular, that means we can solve quadratic equations in the usual way. You can go through the exercise of completing the square if you like, but the quadratic formula works too:



                  $$begin{align}
                  a &= frac{3 + sqrt{(-3)^2 - 4 cdot (-19)}}{2} mod{289 }
                  \ &= frac{3 + sqrt{85}}{2} mod{289}
                  end{align}
                  $$



                  where each choice of square root gives a solution, and conversely, every solution comes from a choice of square root. (for general moduli, it is possible to have more than two square roots, although that can't happen for odd prime powers, such as this problem)



                  To compute square roots, it is usually best to work modulo the prime factors first, then lift to powers of primes, and finally use the Chinese Remainder Theorem to patch the solution together. Here, $289 = 17^2$, so we only have to do the first two steps.



                  So first, we need



                  $$ sqrt{85} = sqrt{0} = 0 mod{17}$$



                  Thus, if $85$ has a square root modulo $289$, it must be a multiple of 17.



                  However,



                  $$ (17c)^2 = 289c^2 = 0 mod{289} $$



                  therefore $85$ does not have a square root modulo $289$. Thus, there is no solution for $a$.







                  share|cite|improve this answer














                  share|cite|improve this answer



                  share|cite|improve this answer








                  edited Nov 20 '15 at 3:08

























                  answered Nov 30 '13 at 15:03









                  HurkylHurkyl

                  111k9119262




                  111k9119262























                      1












                      $begingroup$

                      As $289=17^2,$ let us check the divisibility by $17$



                      $$a^2-3a-19equiv0pmod{289}implies a^2-3a-19equiv0pmod{17}$$



                      $$iff a^2-3aequiv19equiv2iff4a^2-12aequiv8pmod{17}text{ as }(4,17)=1$$



                      $$iff(2a-3)^2equiv0$$



                      $$iff 2aequiv3pmod{17}equiv20iff aequiv10text{ as }(2,17)=1$$



                      So, $a=17c+10$ where $c$ is some integer



                      $$a^2-3a-19=(17c+10)^2-3(17c+10)-19equiv51pmod{289}$$






                      share|cite|improve this answer









                      $endgroup$













                      • $begingroup$
                        Initially , have you taken the expression to be divisible by 17 ?
                        $endgroup$
                        – user2369284
                        Nov 30 '13 at 11:16










                      • $begingroup$
                        @user2369284, yes, I have solved for modulo $17$ as it is a factor of $289$
                        $endgroup$
                        – lab bhattacharjee
                        Nov 30 '13 at 11:18










                      • $begingroup$
                        but how can you take so when it is not given ?
                        $endgroup$
                        – user2369284
                        Nov 30 '13 at 11:19










                      • $begingroup$
                        @user2369284, do you know, if $a$ divides $b$ and $b$ divides $c;a$ must divide $c$. Btw, you can check other answers
                        $endgroup$
                        – lab bhattacharjee
                        Nov 30 '13 at 11:20










                      • $begingroup$
                        @user2369284: No, you don't have to start by solving modulo $17$. However, the arithmetic involved is sometimes simpler if you solve modulo 17, then lift the solution to modulo 289 (e.g. this is often done to calculate square roots). IMO, you shouldn't think about divisibility at all: you should be thinking about modular arithmetic. But if you insist, you know that something divisible by 289 must also be divisible by 17, so it's fair to start there.
                        $endgroup$
                        – Hurkyl
                        Nov 30 '13 at 14:54


















                      1












                      $begingroup$

                      As $289=17^2,$ let us check the divisibility by $17$



                      $$a^2-3a-19equiv0pmod{289}implies a^2-3a-19equiv0pmod{17}$$



                      $$iff a^2-3aequiv19equiv2iff4a^2-12aequiv8pmod{17}text{ as }(4,17)=1$$



                      $$iff(2a-3)^2equiv0$$



                      $$iff 2aequiv3pmod{17}equiv20iff aequiv10text{ as }(2,17)=1$$



                      So, $a=17c+10$ where $c$ is some integer



                      $$a^2-3a-19=(17c+10)^2-3(17c+10)-19equiv51pmod{289}$$






                      share|cite|improve this answer









                      $endgroup$













                      • $begingroup$
                        Initially , have you taken the expression to be divisible by 17 ?
                        $endgroup$
                        – user2369284
                        Nov 30 '13 at 11:16










                      • $begingroup$
                        @user2369284, yes, I have solved for modulo $17$ as it is a factor of $289$
                        $endgroup$
                        – lab bhattacharjee
                        Nov 30 '13 at 11:18










                      • $begingroup$
                        but how can you take so when it is not given ?
                        $endgroup$
                        – user2369284
                        Nov 30 '13 at 11:19










                      • $begingroup$
                        @user2369284, do you know, if $a$ divides $b$ and $b$ divides $c;a$ must divide $c$. Btw, you can check other answers
                        $endgroup$
                        – lab bhattacharjee
                        Nov 30 '13 at 11:20










                      • $begingroup$
                        @user2369284: No, you don't have to start by solving modulo $17$. However, the arithmetic involved is sometimes simpler if you solve modulo 17, then lift the solution to modulo 289 (e.g. this is often done to calculate square roots). IMO, you shouldn't think about divisibility at all: you should be thinking about modular arithmetic. But if you insist, you know that something divisible by 289 must also be divisible by 17, so it's fair to start there.
                        $endgroup$
                        – Hurkyl
                        Nov 30 '13 at 14:54
















                      1












                      1








                      1





                      $begingroup$

                      As $289=17^2,$ let us check the divisibility by $17$



                      $$a^2-3a-19equiv0pmod{289}implies a^2-3a-19equiv0pmod{17}$$



                      $$iff a^2-3aequiv19equiv2iff4a^2-12aequiv8pmod{17}text{ as }(4,17)=1$$



                      $$iff(2a-3)^2equiv0$$



                      $$iff 2aequiv3pmod{17}equiv20iff aequiv10text{ as }(2,17)=1$$



                      So, $a=17c+10$ where $c$ is some integer



                      $$a^2-3a-19=(17c+10)^2-3(17c+10)-19equiv51pmod{289}$$






                      share|cite|improve this answer









                      $endgroup$



                      As $289=17^2,$ let us check the divisibility by $17$



                      $$a^2-3a-19equiv0pmod{289}implies a^2-3a-19equiv0pmod{17}$$



                      $$iff a^2-3aequiv19equiv2iff4a^2-12aequiv8pmod{17}text{ as }(4,17)=1$$



                      $$iff(2a-3)^2equiv0$$



                      $$iff 2aequiv3pmod{17}equiv20iff aequiv10text{ as }(2,17)=1$$



                      So, $a=17c+10$ where $c$ is some integer



                      $$a^2-3a-19=(17c+10)^2-3(17c+10)-19equiv51pmod{289}$$







                      share|cite|improve this answer












                      share|cite|improve this answer



                      share|cite|improve this answer










                      answered Nov 30 '13 at 11:03









                      lab bhattacharjeelab bhattacharjee

                      225k15157275




                      225k15157275












                      • $begingroup$
                        Initially , have you taken the expression to be divisible by 17 ?
                        $endgroup$
                        – user2369284
                        Nov 30 '13 at 11:16










                      • $begingroup$
                        @user2369284, yes, I have solved for modulo $17$ as it is a factor of $289$
                        $endgroup$
                        – lab bhattacharjee
                        Nov 30 '13 at 11:18










                      • $begingroup$
                        but how can you take so when it is not given ?
                        $endgroup$
                        – user2369284
                        Nov 30 '13 at 11:19










                      • $begingroup$
                        @user2369284, do you know, if $a$ divides $b$ and $b$ divides $c;a$ must divide $c$. Btw, you can check other answers
                        $endgroup$
                        – lab bhattacharjee
                        Nov 30 '13 at 11:20










                      • $begingroup$
                        @user2369284: No, you don't have to start by solving modulo $17$. However, the arithmetic involved is sometimes simpler if you solve modulo 17, then lift the solution to modulo 289 (e.g. this is often done to calculate square roots). IMO, you shouldn't think about divisibility at all: you should be thinking about modular arithmetic. But if you insist, you know that something divisible by 289 must also be divisible by 17, so it's fair to start there.
                        $endgroup$
                        – Hurkyl
                        Nov 30 '13 at 14:54




















                      • $begingroup$
                        Initially , have you taken the expression to be divisible by 17 ?
                        $endgroup$
                        – user2369284
                        Nov 30 '13 at 11:16










                      • $begingroup$
                        @user2369284, yes, I have solved for modulo $17$ as it is a factor of $289$
                        $endgroup$
                        – lab bhattacharjee
                        Nov 30 '13 at 11:18










                      • $begingroup$
                        but how can you take so when it is not given ?
                        $endgroup$
                        – user2369284
                        Nov 30 '13 at 11:19










                      • $begingroup$
                        @user2369284, do you know, if $a$ divides $b$ and $b$ divides $c;a$ must divide $c$. Btw, you can check other answers
                        $endgroup$
                        – lab bhattacharjee
                        Nov 30 '13 at 11:20










                      • $begingroup$
                        @user2369284: No, you don't have to start by solving modulo $17$. However, the arithmetic involved is sometimes simpler if you solve modulo 17, then lift the solution to modulo 289 (e.g. this is often done to calculate square roots). IMO, you shouldn't think about divisibility at all: you should be thinking about modular arithmetic. But if you insist, you know that something divisible by 289 must also be divisible by 17, so it's fair to start there.
                        $endgroup$
                        – Hurkyl
                        Nov 30 '13 at 14:54


















                      $begingroup$
                      Initially , have you taken the expression to be divisible by 17 ?
                      $endgroup$
                      – user2369284
                      Nov 30 '13 at 11:16




                      $begingroup$
                      Initially , have you taken the expression to be divisible by 17 ?
                      $endgroup$
                      – user2369284
                      Nov 30 '13 at 11:16












                      $begingroup$
                      @user2369284, yes, I have solved for modulo $17$ as it is a factor of $289$
                      $endgroup$
                      – lab bhattacharjee
                      Nov 30 '13 at 11:18




                      $begingroup$
                      @user2369284, yes, I have solved for modulo $17$ as it is a factor of $289$
                      $endgroup$
                      – lab bhattacharjee
                      Nov 30 '13 at 11:18












                      $begingroup$
                      but how can you take so when it is not given ?
                      $endgroup$
                      – user2369284
                      Nov 30 '13 at 11:19




                      $begingroup$
                      but how can you take so when it is not given ?
                      $endgroup$
                      – user2369284
                      Nov 30 '13 at 11:19












                      $begingroup$
                      @user2369284, do you know, if $a$ divides $b$ and $b$ divides $c;a$ must divide $c$. Btw, you can check other answers
                      $endgroup$
                      – lab bhattacharjee
                      Nov 30 '13 at 11:20




                      $begingroup$
                      @user2369284, do you know, if $a$ divides $b$ and $b$ divides $c;a$ must divide $c$. Btw, you can check other answers
                      $endgroup$
                      – lab bhattacharjee
                      Nov 30 '13 at 11:20












                      $begingroup$
                      @user2369284: No, you don't have to start by solving modulo $17$. However, the arithmetic involved is sometimes simpler if you solve modulo 17, then lift the solution to modulo 289 (e.g. this is often done to calculate square roots). IMO, you shouldn't think about divisibility at all: you should be thinking about modular arithmetic. But if you insist, you know that something divisible by 289 must also be divisible by 17, so it's fair to start there.
                      $endgroup$
                      – Hurkyl
                      Nov 30 '13 at 14:54






                      $begingroup$
                      @user2369284: No, you don't have to start by solving modulo $17$. However, the arithmetic involved is sometimes simpler if you solve modulo 17, then lift the solution to modulo 289 (e.g. this is often done to calculate square roots). IMO, you shouldn't think about divisibility at all: you should be thinking about modular arithmetic. But if you insist, you know that something divisible by 289 must also be divisible by 17, so it's fair to start there.
                      $endgroup$
                      – Hurkyl
                      Nov 30 '13 at 14:54













                      1












                      $begingroup$

                      Inspired by mercio



                      let us find $x,y$ such that $x-y=3,x+y=17implies x=10,y=7$



                      $$a^2-3a-19=(a-10)(a+7)+51$$



                      As $51$ is divisible by $17,$ so must be $(a-10)(a+7)$ to make $17|(a^2-3a-19)$



                      Now $17|(a-10)iff 17|(a+7)$ as $(a+7)-(a-10)=17$



                      So, $17^2|(a-10)(a+7),$ but $17^2not|51$



                      Generalization:



                      Let us find integer $x,y$ such that



                      $displaystyle x-y=-3$(the coefficient of $a$)



                      and $displaystyle x+y=17c$ so that either both or none $(a+x),(a-y)$ will be divisible by $17$



                      $implies 2x=17c-3implies c$ must be odd $=2d+1$(say) for some integer $d$



                      $implies x=17d+7$ and $y=x+3=17d+10$



                      Now, $displaystyle (a+x)(a+y)={a+(17d+7)}{a-(17d+10)}=a^2-3a-(17d+7)(17d+10)$



                      $displaystyleimplies a^2-3a-19={a+(17d+7)}{a-(17d+10)}+(17d+7)(17d+10)-19$



                      $displaystyleimplies a^2-3a-19=underbrace{{a+(17d+7)}{a-(17d+10)}}_{text{terms with difference }=17}+17^2(d^2+d)+51$



                      Now the logic is exactly same as the one above



                      In the above method $d=0$






                      share|cite|improve this answer











                      $endgroup$













                      • $begingroup$
                        why have you taken x-y=3, x+y=17,how did you feel that it was necesary ?
                        $endgroup$
                        – user2369284
                        Nov 30 '13 at 11:23










                      • $begingroup$
                        @user2369284, observe the coefficient of $3$ and we need modulo $17$ first. So that both or none of $a-x,a+y$ be divisible by $17$. See this(math.stackexchange.com/questions/9431/…)
                        $endgroup$
                        – lab bhattacharjee
                        Nov 30 '13 at 11:27












                      • $begingroup$
                        @user2369284, please have a look into the edited version
                        $endgroup$
                        – lab bhattacharjee
                        Nov 30 '13 at 15:52


















                      1












                      $begingroup$

                      Inspired by mercio



                      let us find $x,y$ such that $x-y=3,x+y=17implies x=10,y=7$



                      $$a^2-3a-19=(a-10)(a+7)+51$$



                      As $51$ is divisible by $17,$ so must be $(a-10)(a+7)$ to make $17|(a^2-3a-19)$



                      Now $17|(a-10)iff 17|(a+7)$ as $(a+7)-(a-10)=17$



                      So, $17^2|(a-10)(a+7),$ but $17^2not|51$



                      Generalization:



                      Let us find integer $x,y$ such that



                      $displaystyle x-y=-3$(the coefficient of $a$)



                      and $displaystyle x+y=17c$ so that either both or none $(a+x),(a-y)$ will be divisible by $17$



                      $implies 2x=17c-3implies c$ must be odd $=2d+1$(say) for some integer $d$



                      $implies x=17d+7$ and $y=x+3=17d+10$



                      Now, $displaystyle (a+x)(a+y)={a+(17d+7)}{a-(17d+10)}=a^2-3a-(17d+7)(17d+10)$



                      $displaystyleimplies a^2-3a-19={a+(17d+7)}{a-(17d+10)}+(17d+7)(17d+10)-19$



                      $displaystyleimplies a^2-3a-19=underbrace{{a+(17d+7)}{a-(17d+10)}}_{text{terms with difference }=17}+17^2(d^2+d)+51$



                      Now the logic is exactly same as the one above



                      In the above method $d=0$






                      share|cite|improve this answer











                      $endgroup$













                      • $begingroup$
                        why have you taken x-y=3, x+y=17,how did you feel that it was necesary ?
                        $endgroup$
                        – user2369284
                        Nov 30 '13 at 11:23










                      • $begingroup$
                        @user2369284, observe the coefficient of $3$ and we need modulo $17$ first. So that both or none of $a-x,a+y$ be divisible by $17$. See this(math.stackexchange.com/questions/9431/…)
                        $endgroup$
                        – lab bhattacharjee
                        Nov 30 '13 at 11:27












                      • $begingroup$
                        @user2369284, please have a look into the edited version
                        $endgroup$
                        – lab bhattacharjee
                        Nov 30 '13 at 15:52
















                      1












                      1








                      1





                      $begingroup$

                      Inspired by mercio



                      let us find $x,y$ such that $x-y=3,x+y=17implies x=10,y=7$



                      $$a^2-3a-19=(a-10)(a+7)+51$$



                      As $51$ is divisible by $17,$ so must be $(a-10)(a+7)$ to make $17|(a^2-3a-19)$



                      Now $17|(a-10)iff 17|(a+7)$ as $(a+7)-(a-10)=17$



                      So, $17^2|(a-10)(a+7),$ but $17^2not|51$



                      Generalization:



                      Let us find integer $x,y$ such that



                      $displaystyle x-y=-3$(the coefficient of $a$)



                      and $displaystyle x+y=17c$ so that either both or none $(a+x),(a-y)$ will be divisible by $17$



                      $implies 2x=17c-3implies c$ must be odd $=2d+1$(say) for some integer $d$



                      $implies x=17d+7$ and $y=x+3=17d+10$



                      Now, $displaystyle (a+x)(a+y)={a+(17d+7)}{a-(17d+10)}=a^2-3a-(17d+7)(17d+10)$



                      $displaystyleimplies a^2-3a-19={a+(17d+7)}{a-(17d+10)}+(17d+7)(17d+10)-19$



                      $displaystyleimplies a^2-3a-19=underbrace{{a+(17d+7)}{a-(17d+10)}}_{text{terms with difference }=17}+17^2(d^2+d)+51$



                      Now the logic is exactly same as the one above



                      In the above method $d=0$






                      share|cite|improve this answer











                      $endgroup$



                      Inspired by mercio



                      let us find $x,y$ such that $x-y=3,x+y=17implies x=10,y=7$



                      $$a^2-3a-19=(a-10)(a+7)+51$$



                      As $51$ is divisible by $17,$ so must be $(a-10)(a+7)$ to make $17|(a^2-3a-19)$



                      Now $17|(a-10)iff 17|(a+7)$ as $(a+7)-(a-10)=17$



                      So, $17^2|(a-10)(a+7),$ but $17^2not|51$



                      Generalization:



                      Let us find integer $x,y$ such that



                      $displaystyle x-y=-3$(the coefficient of $a$)



                      and $displaystyle x+y=17c$ so that either both or none $(a+x),(a-y)$ will be divisible by $17$



                      $implies 2x=17c-3implies c$ must be odd $=2d+1$(say) for some integer $d$



                      $implies x=17d+7$ and $y=x+3=17d+10$



                      Now, $displaystyle (a+x)(a+y)={a+(17d+7)}{a-(17d+10)}=a^2-3a-(17d+7)(17d+10)$



                      $displaystyleimplies a^2-3a-19={a+(17d+7)}{a-(17d+10)}+(17d+7)(17d+10)-19$



                      $displaystyleimplies a^2-3a-19=underbrace{{a+(17d+7)}{a-(17d+10)}}_{text{terms with difference }=17}+17^2(d^2+d)+51$



                      Now the logic is exactly same as the one above



                      In the above method $d=0$







                      share|cite|improve this answer














                      share|cite|improve this answer



                      share|cite|improve this answer








                      edited Nov 30 '13 at 15:19

























                      answered Nov 30 '13 at 11:10









                      lab bhattacharjeelab bhattacharjee

                      225k15157275




                      225k15157275












                      • $begingroup$
                        why have you taken x-y=3, x+y=17,how did you feel that it was necesary ?
                        $endgroup$
                        – user2369284
                        Nov 30 '13 at 11:23










                      • $begingroup$
                        @user2369284, observe the coefficient of $3$ and we need modulo $17$ first. So that both or none of $a-x,a+y$ be divisible by $17$. See this(math.stackexchange.com/questions/9431/…)
                        $endgroup$
                        – lab bhattacharjee
                        Nov 30 '13 at 11:27












                      • $begingroup$
                        @user2369284, please have a look into the edited version
                        $endgroup$
                        – lab bhattacharjee
                        Nov 30 '13 at 15:52




















                      • $begingroup$
                        why have you taken x-y=3, x+y=17,how did you feel that it was necesary ?
                        $endgroup$
                        – user2369284
                        Nov 30 '13 at 11:23










                      • $begingroup$
                        @user2369284, observe the coefficient of $3$ and we need modulo $17$ first. So that both or none of $a-x,a+y$ be divisible by $17$. See this(math.stackexchange.com/questions/9431/…)
                        $endgroup$
                        – lab bhattacharjee
                        Nov 30 '13 at 11:27












                      • $begingroup$
                        @user2369284, please have a look into the edited version
                        $endgroup$
                        – lab bhattacharjee
                        Nov 30 '13 at 15:52


















                      $begingroup$
                      why have you taken x-y=3, x+y=17,how did you feel that it was necesary ?
                      $endgroup$
                      – user2369284
                      Nov 30 '13 at 11:23




                      $begingroup$
                      why have you taken x-y=3, x+y=17,how did you feel that it was necesary ?
                      $endgroup$
                      – user2369284
                      Nov 30 '13 at 11:23












                      $begingroup$
                      @user2369284, observe the coefficient of $3$ and we need modulo $17$ first. So that both or none of $a-x,a+y$ be divisible by $17$. See this(math.stackexchange.com/questions/9431/…)
                      $endgroup$
                      – lab bhattacharjee
                      Nov 30 '13 at 11:27






                      $begingroup$
                      @user2369284, observe the coefficient of $3$ and we need modulo $17$ first. So that both or none of $a-x,a+y$ be divisible by $17$. See this(math.stackexchange.com/questions/9431/…)
                      $endgroup$
                      – lab bhattacharjee
                      Nov 30 '13 at 11:27














                      $begingroup$
                      @user2369284, please have a look into the edited version
                      $endgroup$
                      – lab bhattacharjee
                      Nov 30 '13 at 15:52






                      $begingroup$
                      @user2369284, please have a look into the edited version
                      $endgroup$
                      – lab bhattacharjee
                      Nov 30 '13 at 15:52




















                      draft saved

                      draft discarded




















































                      Thanks for contributing an answer to Mathematics Stack Exchange!


                      • Please be sure to answer the question. Provide details and share your research!

                      But avoid



                      • Asking for help, clarification, or responding to other answers.

                      • Making statements based on opinion; back them up with references or personal experience.


                      Use MathJax to format equations. MathJax reference.


                      To learn more, see our tips on writing great answers.




                      draft saved


                      draft discarded














                      StackExchange.ready(
                      function () {
                      StackExchange.openid.initPostLogin('.new-post-login', 'https%3a%2f%2fmath.stackexchange.com%2fquestions%2f586666%2fprove-that-there-is-no-integer-a-for-which-a2-3a-19-is-divisible-by-289%23new-answer', 'question_page');
                      }
                      );

                      Post as a guest















                      Required, but never shown





















































                      Required, but never shown














                      Required, but never shown












                      Required, but never shown







                      Required, but never shown

































                      Required, but never shown














                      Required, but never shown












                      Required, but never shown







                      Required, but never shown







                      Popular posts from this blog

                      Can a sorcerer learn a 5th-level spell early by creating spell slots using the Font of Magic feature?

                      Does disintegrating a polymorphed enemy still kill it after the 2018 errata?

                      A Topological Invariant for $pi_3(U(n))$